Sei sulla pagina 1di 28

Soluzioni a caso

pietro zoppetti
October 2022

1 15/10/2022
1.1 Engel 5-E9 (d)

Si traccino tutte le diagonali di un poligono convesso di n lati denominato


P. Supponendo che nessuna terna di diagonali passi per un punto, si trovi
il numero T di triangoli interni a P.

Per contare tutti i triangoli all’interno del poligono possiamo impostare


l’equazione
Ttot = T0 + T1 + T2 + T3 dove Ti è il numero di triangoli con i vertici coincidenti
con i vertici del poligono

• Possiamo notare che, presi sei vertici distinti del poligono, all’interno del
grafo completo da loro composto viene a crearsi uno e un solo triangolo che
non abbia nessun vertice coincidente con i sei menzionati prima, infatti:
– imposte le condizioni della tesi se si collegano i vertici opposti dell’esagono
i tre segmenti andranno a formare un triangolo interno all’esagono,
in quanto questo sarà convesso
– per formare un triangolo che rispetti queste condizioni le tre diagonali
che lo delimitano (in quanto non può essere delimitato da un lato)
non devono avere vertici in comune
– Un’ovvia condizione, ovvero che le diagonali delimitanti il triangolo
si devono incontrare all’interno dell’esagono, ci porta ad affermare
che non ci può essere più di una di queste che colleghi due vertici che
non siano opposti, ma questo non è possibile a meno che tutte e tre
non siano diagonali con come estremi vertici opposti
Quindi possiamo concludere che T0 = n6 .


• Possiamo adesso ragionare simimilmente per determinare T1 , infatti pos-


siamo osservare che prendendo in considerazione un grafo completo con
come nodi 5 dei vertici del poligono questo avrà al suo interno esattamente
5 triangoli con 1 solo vertice conincidente con i nodi stessi:

1
– per dimostrarlo, consideriamo innanzitutto i 5 triangoli delimitati
da due archi uscenti dallo stesso nodo (che non colleghino due nodi
adiacenti) e dall’unico altro non avente nessun nodo con gli altri due,
e notiamo che possiedono le caratteristiche elencate prima
– è facile dimostrare che non ne esistono altri, dato che la legge di for-
mazione impone che due archi dei tre delimitanti il triangolo debbano
uscire dallo stesso nodo, in quanto altrimenti questo non potrebbe
avere un vertice coincidente con un nodo, e se il terzo arco avesse
nodi in comune con gli altri due avremmo che almeno 2 vertici del
triangolo sarebbero coincidenti con 2 nodi.
– nessun arco può collegare due nodi adiacenti, siccome non si interseca
con nessuna diagonale che non abbia un nodo in comune con questo
stesso
Quindi possiamo concludere che T1 = 5 n5


• Consideriamo quindi T2 ; in questo caso ci basta considerare un quadri-


latero con vertici coincidenti con quelli del poligono:
– Le quattro parti in cui questo è diviso dalle sue due diagonali sono
tutti triangoli delimitati da un lato e due diagonali, che quindi sod-
disfano le caratteristiche dei triangoli che cerchiamo di contare in
T2
– Dato che un quadrilatero non ha 3 diagonali, le alternative per i
segmenti delimitanti il triangolo sono: due lati e una diagonale, che
darebbe origine ad un triangolo con 3 vertici coincidenti con quelli del
poligono, e tre lati, che non darebbe origine ad un triangolo interno
al poligono.

Possiamo quindi concludere che T2 = 4 n4




• Consideriamo adesso l’ultimo caso: banalmente, T3 = n3 in quanto




scegliendo 3 vertici del poligono casualmente questi formeranno sicura-


mente un triangolo che soddisfi la condizione di avere 3 vertici coincidenti
con i vertici del poligono stesso.
Quindi Ttot = n6 + 5 n5 + 4 n4 + n3
   

2
2 18/10/2022
2.1 USAMO 4 2018

Sia p un numero primo, e a1 , . . . , ap dei numeri interi. Dimostrare che


esiste sempre un intero k t.c. i numeri

a1 + k, a2 + 2k, . . . , ap + pk

danno almeno 21 p residui distinti in seguito alla divisione per p.

• Consideriamo due termini della successione, ai , aj : c’è un solo valore di k


compreso nell’intervallo [1, p] per il quale ai + ki ≡ aj + kj.
• Consideriamo quindi una tabella di questo tipo:

k a1 a2 ... ap
1 a1 + 1 a2 + 2 ... ap + p
2 a1 + 2 a2 + 4 ... ap + 2p
3 a1 + 3 a2 + 6 ... ap + 3p
... ... ... ... ...
p a1 + p a2 + 2p ... ap + p2

E per ogni
 riga uniamo con un arco i termini congruenti mod p; risulteremo
avere p2 archi in tutta la tabella.

• Se quindi riusciamo a dimostrare che nel caso in cui gli archi siano dis-
tribuiti equamente per ogni riga in ognuna di queste abbiamo almeno p2
termini distinti dimostriamo la tesi di conseguenza.
• Questo fatto è però certamente vero per ogni p > 2, infatti se distribuiamo
gli archi equamente per ogni riga risultano esserci p−1 2 coppie di termini
uguali per ognuna di queste, ma:
– Immaginiamo quindi che nessuna coppia abbia termini in comune
con le altre in ogni riga: avremmo in tal caso p+1 2 termini distinti
all’interno di ogni riga e la tesi sarebbe dimostrata;
– Si può immaginare adesso che fra gli insiemi di termini uguali in una
riga ce ne sia uno con più di 2 elementi: in tal caso
 il numero di archi
totali nella riga sarebbe ≥ p+1 2+q
≥ 22 , dove q è un

2 in quanto 2
intero maggiore di 0, pertanto la tesi sarebbe dimostrata anche in
questo caso.
• Per quanto riguarda il caso per cui p = 2 la tesi è ovviamente verificata.

3
2.2 USAMO 2022 1

I rettangoli BCC1 B2 , CAA1 C2 , e ABB1 A2 sono costruiti esternamente


ad un triangolo acutangolo ABC. Sapendo che

∠BC1 C + ∠CA1 A + ∠AB1 B = 180◦ ,

dimostrare che le rette B1 C2 , C1 A2 , and A1 B2 concorrono.

La chiave per risolvere questo problema è considerare le circonferenze cir-


conscritte ai tre rettangoli, e ipotizzare che queste siano concorrenti.
Una volta supposto questo non è complicato dimostrarlo, infatti:

• Si consideri la figura sottostante:

• Possiamo determinare l’ampiezza di di ∠ACB, dato che:


– ∠BC1 C + ∠CA1 A + ∠AB1 B = 180◦ ⇐⇒ 180◦ − ∠BC1 C + 180◦ −
∠CA1 A = 180◦ − ∠AB1 B = 180◦ − ∠AA2 B
– abbiamo quindi che AA2 BO è ciclico, ma avendo 3 punti in comune
con il rettangolo ABB1 A2 i due poligoni appartengono alla stessa
circonferenza, per cui la supposizione è dimostrata.
Quanto dimostrato implica che 90◦ = ∠B1 OA = ∠C2 OA = ∠C1 OB = ∠A2 OB =
∠B2 OC = ∠A1 OC, che a sua volta implica che A1 , O, B2 sono allineati, come
anche C2 , O, B1 e A2 , O, C1 , da cui ricaviamo la tesi.

4
3 19/10/2022
3.1 Es. 70 Cap. 13 Halliday

Il raggio Rh di un buco nero è il raggio di una sfera chiamata Orizzonte


degli Eventi, centrata nel buco nero.
Secondo la teoria della relatività generale di Einstein Rh = 2GMc2 , dove
M è la massa del buco nero e c è la velocità della luce.
Supponendo di studiare il buco nero da una distanza pari a 50Rh , e di
non volere che la differenza fra l’accelerazione gravitazionale ai nostri
piedi e quella della nostra testa sia maggiore di 10m/s2 , quale è il limite
per la massa del buco nero?

Il raggio Rh di un buco nero è il raggio di una sfera chiamata Orizzonte degli


Eventi, centrata nel buco nero.
Secondo la teoria della relatività generale di Einstein Rh = 2GM c2 , dove M è la
massa del buco nero e c è la velocità della luce.
Supponendo di studiare il buco nero da una distanza pari a 50Rh , e di non
volere che la differenza fra l’accelerazione gravitazionale ai nostri piedi e quella
della nostra testa sia maggiore di 10m/s2 , quale è il limite per la massa del
buco nero? Considerare la differenza di accelerazione utilizzando la formula
della gravitazione universale non darebbe risultati significativi, in quanto la
differenza di distanza di testa e piedi sarebbe troppo piccola, pertanto dobbiamo
considerare la nostra altezza come un differenziale di distanza dR: per cui la
formula che avremmo usato, ovvero

∆ag = ∆Fg ∆R
si trasforma in
dag = a′ (50Rh ) · dR

Sappiamo che a(x) = G M GM
r 2 ⇒ a (x) = −2 r 3 .
Detto questo, è solo questione di calcolare il risultato.

4 20/10/2022
4.1 BMO 2016 3

Trovare tutti i polinomi monici f a coefficienti interi che soddisfino la


seguente condizione: esiste un intero positivo N t.c. p divide 2(f (p)!) + 1
per ogni primo p > N per cui f (p) è un intero positivo.

Nota: Un polinomio monico ha coefficiente direttivo uguale a 1.

5
• Possiamo affermare che il polinomio f (x) sarà positivo all’interno dell’intervallo
[x0 , +∞) per un certo x0 ∈ R, in quanto il coefficiente direttivo del poli-
nomio è positivo, e la somma algebrica dei termini successivi a quello
n
di
−1
grado massimo in f (x0 ) sarà sicuramente maggiore di −(x0 − 1) xx00 −1 che
a sua volta è minore in valore assoluto rispetto a xn0 considerando x0 − 1
maggiore di tutti i coefficienti del polinomio, dove n = deg{f (x)};
• possiamo inoltre affermare che f (p) < p per qualsiasi p primo tale che
f (p) ∈ Z+ , in quanto altrimenti l’ipotesi non sarebbe verificata; dato che
però questa disuguaglianza deve valere per ogni p maggiore di un certo
valore, deve valere deg{f (x)} ≤ 1, siccome qualsiasi funzione polinomiale
di grado maggiore di 1 con coefficiente direttivo assume valori arbitraria-
mente grandi.
• Abbiamo quindi che f (x) = x − a ∨ 1.
• Analizzando il primo caso, sappiamo che (x − a)! ≡ − 21 (mod p) ≡ (p − 3)!
– L’ultima uguaglianza viene direttamente dal teorema di Wilson, per
cui (p − 1)! ≡ −1(mod p) per ogni primo p; dato che (p − 3)! ≡
(p−1)! 1
(p−2)·(p−1) ≡ − 2

– se esistesse un altro a ̸= −3 che soddisfi l’equivalenza (x − a)! ≡


− 12 (mod p) avremmo che (p−3)(p−4) . . . (p−a+1) ≡ (−1)a−3 (a−1)!
2 ≡
1(mod p) per ogni p maggiore di un certo numero, ma questo è fon-
damentalmente impossibile, in quanto non può esistere un numero
che sia congruente a ±1 modulo p per ogni p maggiore di un certo
valore.
• f (x) = 1 è chiaramente impossibile per le condizioni imposte dall’ipotesi,
pertanto l’unica soluzione è f (x) = x − 3

5 21/10/2022
5.1 Putnam 2001 A5

Trovare tutte le coppie di interi positivi a, n tali che valga la seguente


relazione: an+1 − (a + 1)n = 2001

• Dall’equazione data possiamo facilmente notare che, se n ̸= 0, 2001 ≡


−1 (mod a) pertanto a | 2002 = 2 · 7 · 11 · 13

• Quindi deduciamo che a ̸≡ 0(mod 3), e che di conseguenza a + 1 ̸≡


0(mod 3) in quanto 2001 ≡ 0(mod 3) ⇒ an+1 ≡ (a + 1)n

6
• Da quanto detto prima viene direttamente che a ≡ 1 (mod 3) siccome
altrimenti si raggiungerebbe una contraddizione; questo fatto e l’ultima
equivalenza mostrata nel punto precedente portano a (a+1)n ≡ 1 (mod 3),
che è vero se e solo se n è pari.
• Consideriamo ora l’espressione al primo membro dell’equazione data mod-
ulo a + 1: dato che n è pari, an+1 − (a + 1)n ≡ −1 (mod a + 1), pertanto
a + 1 | 2002
• Fra i sedici divisori di 2002 le uniche coppie di questi che sono consecutivi
sono (1, 2), (13, 14)

– a = 1 non può chiaramente soddisfare l’equazione per qualsiasi n


intero
– a = 13 è una soluzione valida, infatti:
∗ 14n ≡ 1 ∨ 4 ∨ 6 (mod 10) ∧ 13n+1 ≡ 1 ∨ 3 ∨ 9 ∨ 7 (mod 10) dove
14n ≡ 1 (mod 10) ⇐⇒ n = 0
∗ Consideriamo n ≥ 3: 14n ≡ 0 (mod 8) ∧ 13n+1 ≡ 5 (mod 8) il
che è impossibile, dato che 2001 ≡ 1 (mod 8)
∗ Per n ≤ 2 l’unica soluzione valida è ottenuta per n = 2
• Per n = 0 l’unica soluzione è chiaramente a = 2002

Quindi (a, n) = (2002, 0) ∨ (13, 2) sono le uniche due soluzioni.

6 22/10/2022
6.1 British Mathematical Olympiad 1988 4

1 2 3
Per x, y, z ∈ N, risolvere x + y − z =1

• Consideriamo la somma dei primi due termini: sappiamo che 3


z ∈ Q+ , per
cui possiamo affermare che x1 + y2 > 1 ⇐⇒ x1 > y−2
y
y
– Consideriamo quindi y > 2: in tal caso possiamo ottenere x < y−2 =
2
1 + y−2 (1), che è un valore compreso fra 1 e 2 per y ≥ 4
– Possiamo quindi affermare che, dato che nessuno dei tre valori può
essere uguale a 0, per y ≥ 4 x = 1 ⇒ 23 z = y ∀z ∈ Z+ /3 | z ∧ z ≥ 6
che porta a soluzioni sicuramente accettabili per x = 1
– Analizziamo quindi il caso in cui y = 3; per (1), valida anche in
questo caso secondo le condizioni imposte in precedenza, x = 1 ∨ 2:
∗ Per x = 1 otteniamo z = 9
2 che non è una soluzione accettabile.

7
∗ Per x = 2 otteniamo z = 18 che è una soluzione accettabile,
scrivibile come (x, y, z) = (2, 3, 18)
– Per y = 2 otteniamo z = 3x ∀x ∈ Z+ , che è una soluzione ovviamente
valida.
z
– Per y = 1 otteniamo x = 3−z >0⇒0<z<3:
∗ Per z = 1 otteniamo x = 12 , che è una soluzione non accettabile.
∗ Per z = 2 otteniamo x = 2, che porta all’ultima soluzione ac-
cettabile, ovvero (x, y, z) = (2, 1, 2)

7 23/10/2022
7.1 Iran 3rd round 2017 Numbers theory final exam P3

x, y, p, m, n ∈ Z con p primo, m, n positivi e M CD(m, n) = 1. Sapendo


che
xm ≡ y n (mod p)
dimostrare che esiste ed è unico modulo p un intero z t.c.

y ≡ z m (mod p)
x ≡ z n (mod p)

• Per il teorema di Bézout sappiamo che 1 = mk + nq con k, q ∈ Z


• Per dimostrare che, data l’ipotesi, esiste un intero z che soddisfi la tesi,
supponiamo y k xq ≡ z (mod p) per un qualche z ∈ Z; elevando alla m
otteniamo z m ≡ y mk xmq ≡ y mk y nq = y mk+nq = y. Allo stesso modo
elevando alla n otteniamo z n ≡ y nk xnq ≡ xmk xnq = xmk+nq = x, per cui
esiste sempre un intero z che soddisfi la tesi, data l’ipotesi.
• Non resta che dimostrare che z è unico modulo p:
– Supponiamo che esista invece un intero z ′ ̸≡ z (mod p) che sod-
disfi a sua volta la tesi; in tal caso z ′mk ≡ y k (mod p) ∧ z ′nq ≡
xq (mod p) ⇒ z ′mk+nq = z ′ ≡ y k xq (mod p) ≡ z (mod p),
che è una contraddizione della nostra supposizione, quindi la tesi è
dimostrata.

8
7.2 IMO Shortlist 2015 N3

Siano m e n interi positivi t.c. m > n. Si consideri xk = m+k n+k per


k = 1, 2, . . . , n + 1. Dimostrare che se tutti i numeri x1 , x2 , . . . , xn+1
sono interi P = x1 x2 . . . xn+1 − 1 è divisibile per un primo dispari.

Soluzione non attentamente verificata


• Consideriamo M CD(m + k, n + k) = M CD(m − n, n + k) ⇒ n + k |
m − n ∀ k ∈ [1, n + 1] ∩ Z ⇒ 2n | m − n ⇒ n | m ⇒ xk = nq+k
n+k ⇒ q ≡
1 (mod n + k) ⇒ q ≡ 1 (mod 2n) ⇒ q ≡ 1 (mod n)
nq+k

• Quindi, consideriando ogni xk = dk


dk
dk
n+k con dk = M CD(k, n) e m =
dk
nq+k
dk
nq, abbiamo che xk = n+k dove sia numeratore che denominatore sono
dk

coprimi con n, per cui, dato che q ≡ 1 (mod n) abbiamo che xk ≡


1 (mod n)
• Quindi per qualsiasi n ̸= 2r con r ∈ N la tesi è ovviamente verificata.
• Analizziamo adesso il caso in cui n = 2r : sappiamo che m = 2r x quindi
2r + k | 2r (x − 1)
• Supponiamo che la tesi sia falsa: sappiamo che se n = 2r ⇒ P + 1 =
2rQ+1
2r q+k
k=1 2rQ+1 2rQ
+1 2rQ+1
r
2Q +1 2r q+k (2r q+k)− (2r +k)
2rQ
+1 2r q+k dk k=1
r −1
k=1
r −1
k=1
dk 2 2
2r +k = 2rQ
k=1
+1 = 2
2rQ
+1 ⇐⇒ P = 2rQ
2
+1 =
k=1 dk 2r +k 2r +k 2r +k
k=1 k=1 k=1
2rQ r r
+1 22 −1 22 −1
dk
k=1
2rQ
+1 2rQ
+1 r
2rQ
+1
2z ⇒ (2r q + k) − (2r + k) − 2z−2 +1
(2r + k) = 0 ⇐⇒
k=1 k=1 k=1
2rQ
+1 r
2rQ
+1
(2r q + k) = (2z−2 +1
+ 1) (2r + k) ⇐⇒ (q 2r+1 − 1) ⇐⇒
k=1 k=1
r
r r r r r
2P
+1
(q 2 +1
−1−2z−2 +1
)2r(2 +1)
+(q 2r −1−2z−2 +1
)2r·2 k +(q 2r−1 −1−
k=1
r
r r r
2P +1 r
2z−2 +1
)2r(2 −1)
ki ·kj +· · ·+(q−1−2z−2 +1
( (2 +1)!
P
) k ) =
ki ̸=kj ∈Z∩[1,2r +1] k=1
(2r + 1)!
r
• Abbiamo poi che,considerando l’ultima equazione, 22 non divide il sec-
ondo membro, ma divide sicuramente l’ultimo termine al primo mem-
bro, in quanto siamo sicuri, per quanto visto in precedenza, che 2r+1 |
r
q − 1, 2z−2 +1 . Quindi, dato che tutti i termini precedenti sono divisi-
r
bili per potenze di due sicuramente maggiori o uguali a 22 abbiamo una
contraddizione.

9
8 25/10/2022
8.1 Problema del MONT

Dimostrare che i coefficienti della scrittura estesa di (a + b)n dove n è un


intero positivo sono tutti dispari se e solo se n è della forma 2k − 1 per
un qualche intero positivo k.

• Tutti i coefficienti dell’espansione di una potenza di un binomio sono scriv-


ibili come nq con n esponente della potenza del binomio, e q a rappre-


sentare l’esponente della potenza di uno dei due termini del binomio stesso
all’interno di uno dei monomi della scrittura estesa (quindi 0 ≤ q ≤ n).
• Sappiamo che il numero deve essere dispari, in quanto, banalmente, 2r

1 =
2r con r ∈ Z+ , e che inoltre per ogni q pari n · (n − 1) · (n − 2) . . . (n − q + 1)
deve avere come massima potenza di 2 che lo divide la stessa che divide q!
ed in particolare quindi, chiamando 2z la più grande potenza di 2 minore
di n, n · (n − 1) · (n − 2) . . . (n − 2z + 1) deve rispettare la proprietà prima
menzionata, dove q! in questo caso è proprio 2z !.

• Ma siccome questa proprietà deve valere allo stesso modo per q! = (2z −2)!
e quindi per n · (n − 1) · (n − 2) . . . (n − 2z + 3), allora 2z | n − 2z + 1, dato
che n − 2z + 2 è dispari, ma se n − 2z + 1 ≤ n e 2z è la più grande potenza
di 2 minore di n allora n − 2z + 1 = 2z ⇐⇒ n = 2z+1 − 1

9 29/10/2022
9.1 Problema difficult

Si considerino gli interi 0 < x0 < y0 e l’algoritmo



xn+1 = xn +y
2
n
yn+1 = xn+1 yn
Calcolare xn , yn per n → +∞

x2 −y 2 x2 −y 2
• Consideriamo x2n+1 − yn+1
2
= n 4 n ⇒ x2n − yn2 = 04n 0 ⇒ limn→∞ x2n =
2
yn ⇒ limn→∞ xn = yn in quanto siamo sicuri che siano entrambe quantità
positive.
q xn
1+ yn
• Consideriamo poi xyn+1
n+1
= 2 che ricorda la formula trigonometrica
di bisezione del coseno di un angolo, dove possiamo porre xynn = cosα ∧
xn+1 α x0
yn+1 = cos 2 per un certo angolo α, quindi se poniamo y0 = cosθ allora
xn
yn = cos 2θn

10
√ y 2 −x2
• Sappiamo poi che arccos(x) = arcsin 1 − x2 e che ny2 n = 1 − ( xynn )2 e
√ √ n
x2 −y 2 2 −x2
yn y02 −x20
che x2n − yn2 = 04n 0 ⇒ yn
n
= yn ·2n
x √ 2 2 √ 2 2
arccos y0 yn −xn y0 −x0
• Quindi 2n
0
= arccos xn
= arcsin = arcsin x0
yn ·2n ⇒ arccos y0 =
√yn2 2 yn √
y −x y 2 −x2
2n arccos xynn = 2n arcsin ynn n = 2n arcsin yn0·2n 0
√ 2 2
y −x
• L’ultimo membro della catena di uguaglianze per n → +∞ tende a y0n 0
in quanto si può prendere il limite notevole per 21n che tende a 0. Quindi
√ 2 2
y0 −x0
yn = xn = arccos( x0
)
per n → +∞
y0

10 03/11/2022
10.1 Pigeonhole con Kristian

Dato un numero intero positivo n, considerare l’insieme {1, 2, . . . , 2n}, e


se ne prenda un sottoinsieme S di n + 1 elementi; dimostrare che
• In S ci sono almeno due elementi coprimi

• In S c’è almeno un elemento multiplo di un altro.


Se ho n gabbie e n + 1 piccioni e voglio metterli nelle gabbie ci sarà
almeno una gabbia con almeno 2 piccioni

Soluzione
• Proviamo con numeri piccoli: per l’insieme {1,2,3,4} posso prendere S =
{1, 2, 3} ∨ {2, 3, 4} ∨ {3, 4, 1} ∨ {1, 2, 4};
Per l’insieme {1,2,3,4,5,6} S = {1, 2, 3, 4}∨{2, 3, 4, 5}∨{3, 4, 5, 6}∨{1, 3, 4, 5}∨
{1, 4, 5, 6} ∨ {2, 4, 5, 6} ∨ {1, 2, 4, 5} ∨ {1, 2, 4, 6} ∨ {1, 2, 5, 6} ∨ {1, 3, 4, 6} ∨
{1, 3, 5, 6} ∨ {1, 2, 3, 5} ∨ {1, 2, 3, 6} ∨ {2, 3, 5, 6} ∨ {2, 3, 4, 6}

• Notiamo che per ogni S che abbiamo trovato ci sono due numeri consec-
utivi, e quindi coprimi; cerchiamo di dimostrarlo per il caso generale per
assurdo
Nota: la dimostrazione per assurdo consiste nella negazione della tesi,
ovvero ciò che si vuole dimostrare, con l’obiettivo di raggiungere una con-
traddizione dell’ipotesi e quindi un assurdo, determinando quindi una negazione
della negazione della tesi, quindi la tesi risulta vera
– Ipotizziamo che in S possano non esistere elementi consecutivi

11
– Quindi ogni coppia di elementi di S sarà distante di almeno un nu-
mero nell’insieme di partenza
– Da qui possiamo dedurre che sarà necessario lasciare non scelti n
elementi dell’insieme di partenza perché questo accada (dimostrabile
per induzione); ma dovendo sceglierne n + 1 l’insieme di partenza
dovrebbe essere composto da 2n + 1 elementi in totale, ma questo è
ovviamente un assurdo! Infatti gli elementi dell’insieme di partenza
sono per ipotesi 2n
• Quindi, essendo giunti ad una contraddizione della nostra assunzione, pos-
siamo affermare che la prima tesi è vera.

• Dimostriamo ora che la seconda tesi è vera:


– Immaginiamo di ripartire l’insieme di partenza in n sottoinsiemi,
costruiti in questo modo: per ogni numero dispari, costruiamo il
sottoinsieme costituito da tutti gli elementi dell’insieme di partenza
scrivibili come il numero stesso moltiplicato per una potenza di due.
– Possiamo osservare che le intersezioni degli n insiemi cosı̀ determinati
presi a 2 a 2 sono uguali all’insieme vuoto, e l’insieme unione di tutti
è esattamente l’insieme di partenza.
– Risulta quindi che, perché la tesi non sia ovviamente dimostrata, ogni
elemento di S dovrebbe appartenere ad un sottoinsieme differente fra
questi, ma questo è assurdo.

11 06/12/2022
11.1 Il big ritorno con un prob noto

Dimostrare che
P
ϕ(d) = n
d|n

Per n ∈ Z+

11.1.1 Prima soluzione


• Consideriamo le n frazioni
1 2 3 4 n
n, n, n, n, . . . , n

• Notiamo che, riducendo tutte ai minimi termini, al denominatore com-


parirà sempre un divisore di n

12
• Ognuno di questi comparirà ogniqualvolta al numeratore troviamo un nu-
mero della forma k· nd dove k è un numero coprimo con d minore di d stesso;
deduciamo facilmente quindi che ogni divisore di n comparirà esattamente
ϕ(d) volte
• Dato che il numero totale di frazioni è esattamente n la tesi è dimostrata.

11.1.2 Seconda soluzione


Supponiamo che la proprietà valga per tutti i numeri n ∈ Z+ minori di un certo
n ∈ Z e dimostriamola per n:
a
• Scriviamo n = pa1 1 · pa2 2 · pa3 3 . . . pq q come prodotto di fattori primi, e
scegliamo un ni < n uguale a pni

• Possiamo quindi scrivere ϕ(pai i · d) =


P P P P
ϕ(d) = ϕ(d) + ϕ(d) +
n
d|n d|ni d| a d|ni
p i
i

ϕ(pai i ) ni + (pai i − pai i −1 ) · pnai n


P
ϕ(d) = = ni + n − pi = ni − ni + n = n
n i
d| a
p i
i

• Possiamo affermare che questa catena di uguaglianze è vera se e solo se


affermiamo che pai i divide esattamente n, per cui possiamo portare fuori
dalla seconda sommatoria ϕ(pai i ); ipotizzando questo, non resta che di-
mostrare il caso base, per n = 1:
P
– ϕ(d) = ϕ(1) = 1
d|1

Essendo la tesi dimostrata per il caso base, la tesi è dimostrata per in-
duzione per ogni n ∈ Z+

12 08/12/2022
12.1 Dimostrazione che dovevo fare da tempo

Dimostrare che, dati due interi positivi p, q coprimi,


ϕ(p · q) = ϕ(p) · ϕ(q)

• Consideriamo un n ∈ Z della forma n = p11 · p12 · p13 . . . p1k


• Possiamo considerare l’insieme Sn dei numeri che hanno almeno un fattore
primo in comune con n: sottraendo ad n la cardinalità di questo insieme
avremo il valore di ϕ(n).

13
• Per calcolare la cardinalità dell’insieme S possiamo semplicemente sfruttare
il principio di inclusione esclusione:

n
|Sn | = i≤k pni − i,j≤k pin·pj + i,j,d≤k pi ·pnj ·pd −· · ·+(−1)k+1 p1 ·p2 ·p
P P P
3 ...pk

ϕ(n) = n− i≤k pni + i,j≤k pin·pj − i,j,d≤k pi ·pnj ·pd +· · ·+(−1)k p1 ·p2 ·p
n
P P P
3 ...pk

• Consideriamo ora il polinomio monico

P (x) = (x − p1 )(x − p2 )(x − p3 ) . . . (x − pk )

Possiamo notare come ϕ(n) sia, per le relazioni radici-coefficienti, uguale


a

– P (1) se k è pari
– −P (1) se k è dispari
Notiamo che se poi k è pari P (x) può anche essere scritto come P (x) =
(p1 − x)(p2 − x)(p3 − x) . . . (pk − x) mentre se k è dispari P (x) = −(p1 −
x)(p2 − x)(p3 − x) . . . (pk − x)

In ciascuno dei due casi ϕ(n) = (p1 − 1)(p2 − 1)(p3 − 1) . . . (pk − 1)


• Consideriamo ora un numero n′ = n·pa1 1 pa2 2 pa3 3 . . . pakk , e definiamo ϕ(n′ ) =
n′ − |Sn′ | dove Sn′ è l’insieme di numeri ≤ n′ che hanno in comune almeno
un fattore primo con n′ stesso.
Analogamente a prima troviamo la cardinalità di Sn′ utilizzando il prin-
cipio di inclusione-esclusione:

n′ n′ n′ k+1 n′
P P P
|Sn′ | = i≤k pi − i,j≤k pi ·pj + i,j,d≤k pi ·pj ·pd −· · ·+(−1) p1 ·p2 ·p3 ...pk =

= pa1 1 pa2 2 pa3 3 . . . pakk · |Sn |

Da cui

ϕ(n′ ) = pa1 1 pa2 2 pa3 3 . . . pkak ·n−pa1 1 pa2 2 pa3 3 . . . pakk ·|Sn | = pa1 1 pa2 2 pa3 3 . . . pakk (p1 −
1)(p2 − 1)(p3 − 1) . . . (pk − 1)

• Dimostrata la formula generale per la funzione toziente, è immediato ri-


cavare la tesi: difatti, dati due numeri p, q ∈ Z+ , la funzione toziente
del loro prodotto sarà ovviamente il prodotto delle loro rispettive funzioni
toziente.

14
13 09/12/2022
13.1 Dimostro il teorema di Wilson yeee
Teorema di Wilson

Sia p un intero positivo; questo è primo se e solo se (p−1)! ≡ −1 (mod p)

• Dimostriamo che se vale la relazione riportata sopra p è sicuramente primo:

– Consideriamo un qualsiasi p composito: questo per definizione non


potrà essere coprimo con tutti gli interi da 1 a p − 1, per cui (p − 1)! e
p non potranno essere coprimi; è noto però che, se a ≡ b (modc) ⇒
M CD(a, c) | b.

– In questo caso, ipotizzando che la nostra tesi sia falsa avremmo


M CD((p − 1)!, p) | −1 per un qualche p composito (o per p = 1,
che però è ovviamente falso), ma questo è impossibile, dato che ab-
biamo stabilito che M CD((p − 1)!, p) > 1, da cui la tesi.
• Dimostriamo ora che se p è primo la relazione vale sempre:

– Consideriamo l’insieme S = 1, . . . , p − 1, e stabiliamo quali fra i suoi


elementi hanno sé stessi come inverso moltiplicativo modulo p, con p
primo:
∗ Per ogni x ∈ S che abbia sé stesso come inverso moltiplicativo
modulo p vale la relazione
p | x2 − 1 = (x − 1)(x + 1)
Da cui ricaviamo p | x − 1 ∨ x + 1, che porta immediatamente
a x = p − 1 ∨ 1.
– Dimostriamo ora che ogni x appartenente ad S ha un solo inverso
moltiplicativo modulo p appartenente ad S stesso:
∗ Dalla definizione di inverso moltiplicativo sappiamo che per ogni
x ∈ S esisterà un certo y ∈ S t.c. y · x ≡ 1 (mod p)
∗ Ipotizziamo adesso che esista un altro numero y ′ ̸= y ∈ S t.c.
y · x ≡ y ′ · x ≡ 1 (mod p)
∗ In tal caso avremmo y · x − y ′ · x = x · (y − y ′ ) ≡ 0 (mod p), da
cui ricaviamo y − y ′ ≡ 0 (mod p) che è ovviamente assurdo per
quanto ipotizzato, da cui la tesi.

Dai risultati ottenuti abbiamo la tesi, difatti dato che ogni elemento in S di-
verso da p − 1 ed 1 ha un inverso moltiplicativo modulo p diverso da sé stesso
e appartenente ad S stesso possiamo concludere che (p − 1)! ≡p (p − 1) ≡p −1
in quanto consideriamo (p − 1)! come il prodotto di tutti gli elementi di S.

15
14 26/01/2022
14.1 Bello cannare i problemi di Geometria

La circonferenza ω è tangente internamente alla circonferenza Γ nel punto


P.
Si consideri la corda AB su Γ t.c. AB è tangente a ω in C e le corde AP ,
BP intersecano la circonferenza rispettivamente in D, E con DP = 17
e EP = 11.
Determinare la lunghezza di AC.

In figura è rappresentata la configurazione descritta dal problema.

• Notiamo che, per imporre la condizione di tangenza interna, è sufficiente


supporre i centri di ω e Γ (rispettivamente O e O′ ) e il punto P allineati.
• Pertanto, notiamo che ∠O′ P B = ∠OP B = 90◦ − ∠BAP , da cui abbiamo
che, dato che ω è tangente ad AB, ∠CO′ P = 360◦ − 2∠BAP − ∠AP B.
Di conseguenza, ∠CP O′ = ∠BAP + ∠AP 2
B
− 90◦ ⇒ ∠CP B = ∠AP 2
B
,
ovvero P C è una bisettrice.

16
• Da questo possiamo esprimere la condizione di tangenza interna e quella
di tangenza con AB col teorema della bisettrice:
AP /AC = BP /BC [1]
• Inseriamo i dati fornitici in altre tre equazioni:
– La prima equazione sarà ovviamente AC = 196 − BC
– Le altre due esprimono la potenza dei punti A e B rispetto a ω:
AP /AC = AC/AD [2]
BP /BC = BC/BE [3]
• Dalle equazioni [1], [2], [3] otteniamo
AC/AD = BC/BE ⇐⇒ AC/BC = AD/BE [4]
• Ma notiamo che l’equazione [1] può essere riscritta come AC/BC =
AP /BP da cui otteniamo, dalla [4],
AD/BE = AP /BP
che è sufficiente ad affermare AB//DE
• Da cui AP /BP = AC/BC = AC/(196 − AC)17/11 ⇐⇒ AC = 119, che
è la soluzione al quesito.

15 30/01/2022
15.1 Lo ho fatto su un post-it, ma giusto per far finta di
aver fatto qualcosa

Sia ABC triangolo acutangolo scaleno e P la proiezione di B sull’asse


di AC, M è il punto medio di BC, Q è l’intersezione tra AB e P M ,
dimostrare che BP Q è isoscele.

• Rispetto agli altri problemi questo è molto più easy: si prolunghi P M fino
a che non interseca AC in K.
• Notiamo che i triangoli △M KC, △M P B sono congruenti, valendo ovvi-
amente AC ∥ BP
• Pertanto, dato che, chiamando R la proiezione di P su AC, AR = RC =
RK + KC = AZ + ZR dove Z è la proiezione di B su AC.
Da cui AZ = KR per cui AKBP è un trapezio isoscele
• Da questo si ha la tesi: infatti si verifica facilmente che ∠BP Q = ∠P BQ =
∠CAB

17
16 31/01/2023
16.1 Da oggi scrivo almeno un problema qui ogni giorno
v Anche questo non è difficile, ma ci ho messo un po’ e lo ho trovato carino.
• Chiamiamo α l’angolo ∠ACB: notiamo che la retta AC biseca BH dato
che su di essa giace il diametro perpendicolare al segmento.
Pertanto, tracciando CH notiamo che ∠HCA = α
• Ma ∠BGH e ∠BCH insistono su BH, per cui ∠XGC = ∠CAB = 180◦ −

• Da cui abbiamo che, chiamando M il punto medio di AC,
M P · R/2 = GM · M B
• Sappiamo però, che GCBK, dove K è la seconda intersezione della retta
AC con ω, è ciclico, per cui abbiamo GM · M B = 34 R2 da cui, sostituendo
nell’equazione precedente, abbiamo M C = 32 R, che implica la tesi.

17 23/02/2023
17.1 Non lo ho chiuso, perché stavo a cercare un centro
di omotetia

Sia ABC un triangolo acutangolo e sia K un punto sulla sua circon-


ferenza circoscritta, dall’altra parte di A rispetto a BC.
Sia M la riflessione di K attraverso BC ed L la riflessione di K attraverso
AB.
Sia poi H l’ortocentro di ABC e E il punto di intersezione diverso da
B tra la circonferenza circoscritta ad ABC stesso e quella circoscritta a
BLM .
Si dimostri che EM , BC, KH sono concorrenti.

• Si considerino le riflessioni di H attraverso BC e AB, rispettivamente HA


e HC : HA HM K è allora certamente un trapezio isoscele, pertanto HK,
BC, e M HA concorrono.
La tesi si riduce quindi a dimostrare che HA , M ed E sono allineati.
• Consideriamo quindi il punto E ′ allineato con M e HA e dimostriamo che
questo è conciclico con B, L, M :
– Questa riformulazione della tesi implica ∡LBM = ∡LE ′ M (directed
angles maledetti, ma sono comodi)
– Ma BC è bisettrice di ∠M BK ed AB è bisettrice di ∠LBM , per cui
∡LBM = ∡LBK + ∡KBM = 2∡ABC

18
– Da cui la tesi è equivalente a ∡LE ′ M = 2∡ABC
– Supponiamo ora che HC , L ed E ′ siano allineati: se questo fosse vero
avremmo la tesi equivalente a dimostrare ∡HC BHA = 2∡ABC che
è ovviamente vero per simmetria.
– Dimostrare che i tre punti sono allineati è equivalente a mostrare
∠KLHC = ∠CHC E ′ = ∠E ′ HA C = ∠M HA C.
Ma ∠KLHC = ∠LKH, e ∠LKH = ∠M HA C è ovviamente verifi-
cata
Da cui la tesi.

18 25/02/2023
18.1 IMO Shortlist 2000 G3

Riflettiamo H attraverso i lati e definiamo D, E, F come in figura: che succede?

• Riflettiamo H attraverso i lati del triangolo, per trovare HA , HB , HC come


in figura; è noto che questi tre punti si trovino sulla circonferenza circo-
scritta ad ABC.

19
• Notiamo che per dimostrare la tesi è sufficiente trovare tre punti che rispet-
tino le condizioni imposte
• Definiamo quindi D, E, F come i punti di intersezione dei raggi della cir-
conferenza circoscritta ad ABC che congiungono il circocentro del trian-
golo e una delle riflessioni coi lati del triangolo stesso, come in figura.
Non resta ora che dimostrare che le ceviane AD, BE, CF concorrono:
– Si nota come, essendo H il coniugato isogonale di O,
∠OBHC = ∠OHC B = ∠ABC = ∠OHA B
e analogamente
∠CHB E = ∠CHA D = ∠BCA
e
∠AHB E = ∠AHC F = ∠BAC
– Da cui, utilizzando il teorema dei seni sui triangoli △BHC F , △BHA D,
△CHA D, △CHB E, △AHB E, △AHC F per trovare i segmenti BF ,
AF , CD, DB, CE, AE in funzione dei segmenti HC F , HA D, HB E si
può constatare che l’uguaglianza descritta dal teorema di Ceva come
condizione necessaria e sufficiente affinché tre ceviane concorrano è
sempre verificata.

Da cui la tesi

19 29/03/2023
19.1 Giusto per scrivere qualcosa

Si dimostri che sono infiniti i numeri primi della forma 4k − 1

Dimostriamo la tesi per assurdo:

• Notiamo che esiste almeno un primo che rispetta le condizioni, ovvero 3


• Supponiamo ora che i primi di questo tipo siano in un numero finito mag-
giore o uguale a uno; da cui abbiamo un insieme finito S = {p1 , . . . , pk }
che contiene tutti i k numeri con le caratteristiche sopracitate.
• Notiamo che di certo tutti questi sono coprimi con 4 per definizione

• Da cui, se consideriamo il prodotto P di tutti gli elementi di S abbiamo


per ipotesi P ≡ ±1 (mod 4).
• Da cui abbiamo P + 2 ∨ P + 4 ≡ −1 (mod 4)

20
• Abbiamo quindi che P + 2 ha almeno un primo nella sua fattorizzazione
congruo a −1 modulo 4.
• Abbiamo però per costruzione M CD(P + 2, P ) = 1, da cui abbiamo un
assurdo e quindi la tesi.

20 06/04/2023
20.1 POTD 1473

Sia ABC un triangolo isoscele di base AB, e si prenda un punto D ∈ AB


t.c. AD < AB.
Si chiamino P, Q le proiezioni di D su BC, AC rispettivamente, e si
definisca E = r ∩ QC dove r è l’asse di P Q.
Si chiami invece F il punto di intersezione diverso da C delle circonferenze
circoscritte ad ABC e a P QC.
Supponendo P, E, F allineati dimostrare ∠BCA = 90◦ .

• QP CF è un quadrilatero ciclico, e per costruzione EP = EQ per con-


gruenza di triangoli.
Da cui ∠P QC = ∠F P Q = ∠F CQ, che ci porta ad affermare che il quadri-
latero di cui sopra è un trapezio, e pertanto un trapezio isoscele dalla con-
dizione di ciclicità.

• Abbiamo poi che anche ABF C è ciclico, da cui ricaviamo ∠F BC =


∠F AC. Inoltre, dato che ∠AF B = ∠ACB = ∠QF P , abbiamo ∠BF P =

21
∠AF Q per differenza di angoli sicuramente congruenti.

• Da questo abbiamo la similitudine △AQF ∼ △BP F ed in particolare


BP AQ
P F = QF e, dato che l − AQ = QC = P F ∧ l − BP = P C = QF detto
l = AC = BC, riscriviamo la proporzione come

l−BP AQ
l−AQ = BP ⇐⇒
AQ 2(AQ−BP )
⇐⇒ BP = l+AQ−BP
l+BP −AQ = 1 + l−(AQ−BP ) ⇒
AQ AQ
⇒ l BP − (AQ − BP ) BP = l + (AQ − BP ) ⇐⇒
(AQ−BP )l
AQ AQ
( BP + 1)(AQ − BP ) = l( BP − 1) ⇒ (AQ − BP ) = (AQ+BP
BP
) ⇒
BP
⇒ AQ + BP = l

Da cui ricaviamo che il triangolo ABC è rettangolo, che è la tesi; infatti:


– Per ogni triangolo rettangolo e per ogni punto D sulla sua ipotenusa
vale certamente questa uguaglianza
– Supponendo di avere un triangolo per cui questa uguaglianza sia vera
dato un certo punto D sulla sua base abbiamo che aumentando la dis-
tanza del vertice dalla base ma mantenendo la proiezione dello stesso
sulla base nello stesso punto i lati obliqui del triangolo aumenteranno
di lunghezza, e il coseno degli angoli alla base diminuirà; viceversa
avvicinando il vertice alla base.

– Da cui può esistere un solo triangolo con punto D e date proiezione


del vertice sulla base e lunghezza della stessa che soddisfi l’equazione
di cui sopra.

– Ma dato che date queste tre ipotesi si ha sempre uno e un solo tri-
angolo rettangolo che soddisfa la stessa abbiamo la proposizione che
dimostra la tesi.

22
21 08/04/2023
21.1 POTD 1474

Determinare per quali valori di n prendendo un punto su ogni lato di


un n-agono regolare diverso dai vertici ottengo un nuovo poligono non
regolare ma con tutti gli angoli uguali.

• La somma degli angoli interni di un n-agono è uguale a (n − 2) · 180◦ , da



cui ogni angolo del nuovo n-agono dovrà misurare n−2
n · 180 .

• Considero tre punti su tre lati consecutivi, e traccio i due lati del nuovo
poligono da loro definiti: i triangoli formati da due di questi punti consec-
utivi ed il vertice fra di loro saranno tutti simili, per differenza di angoli
sicuramente congruenti.
In particolare è condizione necessaria che, numerati i vertici del poligono
originario da V1 a Vn e i vertici del nuovo poligono da P1 a Pn come
nell’esempio in figura, tutti gli angoli del tipo ∠Vi Pi Pi−1 con i consider-
ato modulo n siano della stessa ampiezza.
• Considero il caso in cui n è pari: ogni valore che rispetta questa condizione
è accettabile, infatti basterà, preso un punto P1 , prendere il successivo sul
lato consecutivo coincidente col suo simmetrico rispetto alla bisettrice. I
triangoli che si verranno a formare saranno tutti simili in quanto isosceli

23

con un angolo ampio n−2 n · 180 , e per differenza di angoli certamente con-
gruenti ogni angolo interno al nuovo poligono sarà della stessa ampiezza.
Siamo poi certi che ciò che si viene a creare sia una linea spezzata chiusa
proprio perché il numero di lati è pari, e pertanto il simmetrico rispetto
alla bisettrice dell’ultimo punto scelto procedendo in senso antiorario sarà
proprio P1 ; se invece n fosse stato dispari il punto cosı̀ costruito sarebbe
stato coincidente col simmetrico di P1 rispetto al punto medio del lato su
cui quest’ultimo giace.
Per poi evitare che il nuovo poligono non sia effettivamente regolare basta
ovviamente scegliere un punto P1 non coincidente con il punto medio del
lato su cui questo giace, affinché i triangoli considerati siano simili ma non
tutti congruenti.
• Considero il caso in cui n è dispari: come affermato sopra non è possibile
ripetere la costruzione illustrata. Per quanto affermato in precedenza,
però, chiamata ai la lunghezza del segmento Vi Pi abbiamo che il successivo
in questa forma sarà lungo l − kai dove l = Vi Vi+1 e k = Pi−1 Vi
ai , che è una
costante (vedi sopra).
Da cui
i−2
ai = (−k)i−1 a1 + (−k)x · l ⇒
P
x=0
n+1−2
⇒ a1 = (−k)n+1−1 a1 + (−k)x · l
P
x=0

Dato che a1 è il n + 1-esimo segmento ottenuto con questo processo.

• Sostituiamo n = 2m + 1 e otteniamo:
2m
a1 = (−k)2m+1 a1 + (−k)x · l ⇐⇒
P
x=0
1+k2n+1
⇐⇒ a1 = 1+k · l − k 2n+1 · a1 ⇐⇒
2n+1
1+k 2n+1 l
⇐⇒ 1+k · l − (k + 1) · a1 = 0 = 1+k − a1 ⇐⇒
⇐⇒ a1 + ka1 = l

Ma questo è impossibile per ipotesi, dato che altrimenti otterremmo ogni


triangolo Vi Pi Pi−1 congruente a ognuno degli altri in questa forma, da cui
otterremmo un poligono regolare.

Da cui n = 2m è l’unico caso accettabile per ogni m naturale positivo.

24
21.2 Cese 3 2020

Let a1 , a2 , . . . , a2020 and b1 , b2 , . . . , b2020 be real numbers(not necessarily


distinct). Suppose that the set of positive integers n for which the fol-
lowing equation: |a1 |x−b1 |+a2 |x−b2 |+· · ·+a2020 |x−b2020 || = n [1] has
exactly two real solutions, is a finite set. Prove that the set of positive
integers n for which the equation [1] has at least one real solution, is also
a finite set.

• Definiamo f (x) = |a1 |x − b1 | + a2 |x − b2 | + · · · + a2020 |x − b2020 ||


• Notiamo che, per x < −|max{bi }| la funzione è una retta in valore asso-
luto, e quindi nell’intervallo x ∈ (−∞, −|max{bi }|) la funzione presenta
al massimo un punto angoloso, e pertanto al massimo un punto di inver-
sione.
Possiamo fare poi lo stesso ragionamento per x > |max{bi }| da cui ri-
caviamo che nell’intervallo x ∈ (|max{bi }|, +∞) la funzione presenta al
massimo un punto di inversione.
• Troviamo facilmente poi che lim |x − bi | = lim x
x→±∞ x→+∞

• Da questo abbiamo che lim f (x) = lim f (x)


x→+∞ x→−∞

• Abbiamo che la funzione è certamente limitata inferiormente essendo in


valore assoluto, ed essendo continua su tutto R affinché sia limitata anche
superiormente è necessario e sufficiente che il limite per x → ±∞ di f (x)
non tenda ad infinito.
• Se però la funzione fosse limitata sia superiormente che inferiormente
avremmo la tesi, infatti l’equazione ammetterebbe in generale soluzioni
in R per un insieme finito di interi n dato che l’equazione non può ovvi-
amente ammettere soluzioni in R per valori al di fuori dell’immagine di
f (x).
• Abbiamo però che, se lim f (x) tende all’infinito, l’ipotesi non è verificata,
x→∞
infatti:
– Immaginiamo di prendere il grafico della funzione per ogni suo valore
superiore a quelli di tutti i suoi punti di inversione: abbiamo che per
ogni valore intero n in questo intervallo la retta di equazione y = n
intersecherà il grafico della funzione in esattamente due punti, da
cui per ogni n intero in questo intervallo abbiamo che l’equazione
f (x) = n restituisce esattamente due soluzioni reali.
– Essendo tuttavia l’intervallo un intervallo illimitato gli n interi all’interno
di questo sono infiniti, da cui abbiamo un assurdo.

25
• Dato che abbiamo concluso che l’unico caso favorevole è per x → ±∞ di
f (x) non tendente ad infinito ed abbiamo dimostrato la tesi per lo stesso
abbiamo la tesi.

21.3 Principio di identità dei polinomi

Siano p, q due polinomi: p(x) coincide con q(x) se e solo se p(α) = q(α)
per ogni α ∈ C

• Siano a1 , a2 , a3 , . . . , an le radici di p(x):queste saranno, supponendo p(α) =


q(α) ∀α, anche le radici di q(x).
Sappiamo inoltre che per lo stesso motivo non esisteranno altre radici di
q(x), da cui possiamo scrivere

p(x) = a · (x − x1 ) · · · (x − xn )
q(x) = b · (x − x1 ) · · · (x − xn )

Ma per ogni valore di α t.c. p(x) = q(x) ̸= 0 abbiamo che certamente


allora p(α) q(α)
a = b ma dato che p(α) = q(α) per quanto supposto abbiamo
a = b, e pertanto abbiamo che p(x) e q(x) coincidono.
• Siano p(x) e q(x) coincidenti: è allora vero che p(x) = q(x) = a · (x −
x1 ) · · · (x − xn ) da cui p(α) = a · (α − x1 ) · · · (α − xn ) e q(α) = a · (α −
x1 ) · · · (α − xn ), da cui p(α) = q(α) ∀α ∈ C.
• Avendo dimostrato entrambe le direzioni della doppia implicazione abbi-
amo la tesi.

21.4 Assegnazione di n + 1 valori

Assegnate n + 1 coppie di numeri


(a0 , b0 ), (a1 , b1 ), . . . , (an , bn ) , ai ̸= aj

Esiste ed è unico un polinomio p(x) di grado ≤ n t.c. p(ai ) = bi ∀i ∈


{0, . . . , n}

Dimostriamo la tesi per induzione:


• Passo base: Notiamo che, per n = 0, abbiamo la tesi: difatti se il grado
del polinomio p(x) deve essere ≤ 0 il polinomio sarà costante, ovvero della
forma p(x) = k con k costante, dato che non esistono polinomi di grado

26
diverso da un intero nonnegativo.
Dato che poi p(a0 ) = b0 avremo p(x) = b0 come unica soluzione per quanto
affermato in precedenza.
• Passo induttivo: Supponiamo la tesi vera per ogni insieme di n+1 coppie
di valori {(a0 , b0 ), (a1 , b1 ), . . . , (an , bn )} e dimostriamola per ogni insieme
di n + 2 coppie di valori {(a0 , b0 ), (a1 , b1 ), . . . , (an , bn ), (an+1 , bn+1 )}:
– Notiamo come ogni insieme
S = {(a0 , b0 ), (a1 , b1 ), . . . , (an , bn ), (an+1 , bn+1 )}
di n + 2 coppie di valori sia generato dall’unione di un insieme
A = {(a0 , b0 ), (a1 , b1 ), . . . , (an , bn )}
di n+1 coppie di valori e di un insieme B = {(an+1 , bn+1 )} contenente
solo una coppia di valori; supposto che esista un polinomio p(x) che
soddisfi la condizione imposta dalla tesi per l’insieme A, allora se
dovesse esistere un polinomio che la soddisfi anche per l’insieme S
questo sarebbe certamente della forma
Q(x) = p(x) + r(x) [1]
Dove r(x) è un polinomio di grado ≤ n + 1
– Notiamo che, per la [1], considerato un certo ai appartenente ad
una delle coppie incluse in A = {(a0 , b0 ), (a1 , b1 ), . . . , (an , bn )} ⊂ S
abbiamo Q(ai ) = p(ai ) + r(ai ) = bi + r(ai ) = bi da cui r(ai ) = 0 per
ogni ai appartenente ad una delle coppie incluse in A.
Q
– Da cui r(x) = k (x − ai ) con k costante dato che r(x) ha
i∈{0,...,n}
grado ≤ n + 1.
Q
– Ma allora, preso p(an+1 ) = h e (an+1 − ai ) = l ovviamente
i∈{0,...,n}
costanti note, abbiamo Q(an+1 ) = h + k · l = bn+1 da cui k è uni-
vocamente determinato, a meno che an+1 non sia uguale ad un altro
ai ; solo in quel caso abbiamo l = 0 e k indeterminato o incognita di
un’equazione impossibile, ma questo per ipotesi non può accadere.
Da cui la tesi

• Concludiamo cosı̀ che per ogni n ≥ 0 la tesi è verificata.

27
21.5 Divisione Euclidea

Siano a(x) e b(x) due polinomi, con deg(a(x)) > 0. Allora esistono e
sono unici due polinomi q(x) e r(x) tali che:

(a) b(x) = a(x) · q(x) + r(x)


(b) r(x) = 0 oppure deg(r(x)) < deg(a(x))

• Prendiamo b(x) = a(x) · l1 (x) + b1 (x) e b1 (x) = a(x) · l2 (x) + b2 (x) se


questo è vero ricaviamo facilmente b(x) = a(x) · (l1 (x) + l2 (x)) + b2 (x). in
Pk
generale, ricaviamo b(x) = a(x) · ( li (x)) + bk (x) dove
i=1

ni m
• Supponiamo bi (x) = bi,j · xj e a(x) = aj · xj : fintanto che ni > m
P P
j=0 j=0
possiamo moltiplicare a(x) per li−1 (x) =

28

Potrebbero piacerti anche